Is My Approach Correct? Solving the Integral of tan^4x-sec^4x

  • Thread starter Thread starter mathnoobie
  • Start date Start date
  • Tags Tags
    Integrating
Click For Summary
SUMMARY

The integral of tan^4(x) - sec^4(x) can be approached by separating it into two distinct integrals: ∫tan^4(x)dx and ∫sec^4(x)dx. The user initially derived the solution (tan^3(x))/3 - 2tan(x) - x - (tan^2(x))/2, which does not equate to the simpler solution of x - 2tan(x). It is recommended to simplify tan^4(x) - sec^4(x) before attempting integration for a more straightforward solution.

PREREQUISITES
  • Understanding of integral calculus
  • Familiarity with trigonometric identities
  • Knowledge of differentiation techniques
  • Experience with simplifying algebraic expressions
NEXT STEPS
  • Learn techniques for simplifying trigonometric expressions before integration
  • Study the integration of secant and tangent functions
  • Explore the use of substitution methods in integral calculus
  • Practice differentiating complex functions to verify integration results
USEFUL FOR

Students and professionals in mathematics, particularly those focusing on calculus, trigonometry, and integral solutions.

mathnoobie
Messages
62
Reaction score
0

I looked online and it gave a really simple method to solve it and it ended up with the solution
x-2tan(x)
I took a really weird approach and I'm not sure if my answer is right, and I'm hoping if someone could take the same approach and confirm if I did it correctly.
I basically separated the integral into 2 separate integrals and got a solution.
∫tan(^4)xdx-∫sec(^4)xdx

When I was done, I got the answer of
(tan(^3)x)/3-2tanx-x-(tan(^2)x)/2
 
Physics news on Phys.org
You can always differentiate your answer and see if you get the original integrand.
However, the answer you got isn't equivalent to the answer x - 2tan x.
 
You might want to try simplifying tan^4x-sec^4x first.
 
Question: A clock's minute hand has length 4 and its hour hand has length 3. What is the distance between the tips at the moment when it is increasing most rapidly?(Putnam Exam Question) Answer: Making assumption that both the hands moves at constant angular velocities, the answer is ## \sqrt{7} .## But don't you think this assumption is somewhat doubtful and wrong?

Similar threads

  • · Replies 2 ·
Replies
2
Views
3K
  • · Replies 8 ·
Replies
8
Views
2K
Replies
3
Views
2K
Replies
2
Views
1K
  • · Replies 4 ·
Replies
4
Views
2K
  • · Replies 22 ·
Replies
22
Views
3K
  • · Replies 5 ·
Replies
5
Views
1K
  • · Replies 1 ·
Replies
1
Views
10K
  • · Replies 6 ·
Replies
6
Views
1K
  • · Replies 3 ·
Replies
3
Views
2K